2002 AMC 10P Problems/Problem 5

Revision as of 20:49, 14 July 2024 by Wes (talk | contribs) (Solution 1)

Problem

Let $(a_n)_{n \geq 1}$ be a sequence such that $a_1 = 1$ and $3a_{n+1} - 3a_n = 1$ for all $n \geq 1.$ Find $a_{2002}.$

$\text{(A) }666 \qquad \text{(B) }667 \qquad \text{(C) }668 \qquad \text{(D) }669 \qquad \text{(E) }670$

Solution 1

The recursive rule is equal to $a_{n+1}=\frac{1}{3}+a_{n}$ for all $n \geq 1.$ By recursion, $a_{n+2}=\frac{1}{3}+a_{n+1}=$a_{n+2}=\frac{1}{3}+\frac{1}{3}+a_n.$If we set$n=1$and repeat this process$2001$times, we will get$a_{2001+1}=\frac{1}{3}(2001) + a_1=667+1=668.$Thus, our answer is$\boxed{\textbf{(C) } 668}.$

See also

2002 AMC 10P (ProblemsAnswer KeyResources)
Preceded by
Problem 4
Followed by
Problem 6
1 2 3 4 5 6 7 8 9 10 11 12 13 14 15 16 17 18 19 20 21 22 23 24 25
All AMC 10 Problems and Solutions

The problems on this page are copyrighted by the Mathematical Association of America's American Mathematics Competitions. AMC logo.png